Aut=Inn

Neue Frage »

Gast11022013 Auf diesen Beitrag antworten »
Aut=Inn
Meine Frage:
Zeigen Sie



Ich habe folgende Lösung gefunden, die ich gerade versuche zu verstehen und die ich hier mal aufschreibe.

Meine Ideen:
I.




II.

(Automorphismus) ist bestimmt durch die Bilder von .

Weil Automorphismus ist, muss
, da als Automorphismus ordnungserhaltend ist.







1. Frage:
Sehe ich das richtig, dass man bei I. den Homomorphiesatz verwendet und das für den Isomorphismus gilt, dass ?

2. Frage:
Kann man immer die Ordnung von bestimmen mit ?

3. Frage:
Warum müssen die Bilder in der Menge liegen? Was ist mit ordnungserhaltend gemeint?

(Meine Idee: Ein Automorphismus ist ja bijektiv. Deswegen wird die Ordnung natürlich erhalten.]
Wieso ist ? Das bedeutet doch, dass es 6 bijektive Homomorphismen von nach gibt. Aber wie weiß man, dass es gerade sechs sind?


Kann man sogar sagen, dass ein Automorphismus hier z.B. Zweierzykel wieder auf Zweierzykel abbildet und den einzigen Dreierzykel auf den einzigen Dreierzykel, also auf sich selbst? Wenn ja, wieso?


Danke für jede Hilfe!
Mystic Auf diesen Beitrag antworten »
RE: Aut=Inn
Zitat:
Original von Dennis2010
Weil Automorphismus ist, muss
, da als Automorphismus ordnungserhaltend ist.





Aus logischer Sicht ist das sehr "wirr" angeschrieben und macht für mich keinen Sinn... Richtig wäre:



Ad Frage 1,2: Ja.

Zitat:


3. Frage:
Warum müssen die Bilder in der Menge liegen? Was ist mit ordnungserhaltend gemeint?

(Meine Idee: Ein Automorphismus ist ja bijektiv. Deswegen wird die Ordnung natürlich erhalten.]

Hm, wenn für dich das ohnehin "natürlich" ist, warum fragst du dann? verwirrt

Zitat:

Kann man sogar sagen, dass ein Automorphismus hier z.B. Zweierzykel wieder auf Zweierzykel abbildet und den einzigen Dreierzykel auf den einzigen Dreierzykel, also auf sich selbst? Wenn ja, wieso?

Da Automorphismen "ordnungserhaltend" sind, schon wieder vergessen? unglücklich Und es gibt nur einen einzigen Dreierzyklus? Bist du dir da sicher? verwirrt
Gast11022013 Auf diesen Beitrag antworten »
RE: Aut=Inn
Dann habe ich nicht verstanden, was man mit "ordnungserhaltend" meint.

Ich dachte, das bedeutet: Wenn eine Gruppe z.B. 5 Elemente hat, dann hat die Bildmenge auch wieder 5 Elemente.

Aber anscheiend sagt das etwas aus über die Ordnung der einzelnen Elemente und ihrer Bilder?
Mystic Auf diesen Beitrag antworten »
RE: Aut=Inn
Zitat:
Original von Dennis2010
Aber anscheiend sagt das etwas aus über die Ordnung der einzelnen Elemente und ihrer Bilder?

Genau! Freude
Gast11022013 Auf diesen Beitrag antworten »
RE: Aut=Inn
Aber was sagt es darüber aus? Big Laugh



In einem anderen Thread hatte ich gefragt:
"Ein Gruppenhomomorphismus erhält doch auch die Zykellängen und den Zykeltyp... oder? "


Das schien absurd zu sein.

Aber genau das passiert doch hier??
Mystic Auf diesen Beitrag antworten »
RE: Aut=Inn
Dass unter einem Automorphismus stets und die gleiche Ordnung haben, oder was meinst du?

Edit: Ist für dich "ordnungserhaltend" dasselbe wie die "Zykellänge" und den "Zykeltyp" erhaltend? Dann schau dir mal die zwei Permutationen (12) und (12)(34) in genauer an... geschockt
 
 
Gast11022013 Auf diesen Beitrag antworten »
RE: Aut=Inn
Nochmal kurz für meine eigene Klarheit:

Es ist falsch, dass ein Gruppenhomomorphismus Zykellänge und Zykeltyp erhält. Korrekt?



Aber es ist richtig, dass JEDER Gruppenhomomorphismus die Ordnung eines Elements erhält?

Wirklich für JEDEN Gruppenhomomorphismus oder nur für Automorphismen?


Edit:

Ich habe die Begriffe "Ordnung" und "Zykellänge" nicht richtig unterschieden, danke für den wichtigen Hinweis!
Mystic Auf diesen Beitrag antworten »
RE: Aut=Inn
Zitat:
Original von Dennis2010
Nochmal kurz für meine eigene Klarheit:

Es ist falsch, dass ein Gruppenhomomorphismus Zykellänge und Zykeltyp erhält. Korrekt?

Es ist korrekt, dass es falsch ist, ja...

Zitat:
Original von Dennis2010
Aber es ist richtig, dass JEDER Gruppenhomomorphismus die Ordnung eines Elements erhält?

Oh Gott, hast du z.B. noch nie den Gruppenhomomorphismus gesehen, der alles auf das Einselement abbildet? Erhält dieser die Ordnungen der Elemente? geschockt
Gast11022013 Auf diesen Beitrag antworten »
RE: Aut=Inn
Ich habe den noch nie gesehen, dieser Gruppenhomomorphismus erhält die Ordnungen dann aber nicht.



Wolltest Du damit mitteilen, dass das also nur für Automorphismen gilt?
Mystic Auf diesen Beitrag antworten »
RE: Aut=Inn
Zitat:
Original von Dennis2010
Wolltest Du damit mitteilen, dass das also nur für Automorphismen gilt?

Ich dachte, das ist nach allem, was ich dazu schon gesagt habe, bereits klargestellt, aber wenn du noch eine Bestätigung brauchst: JA!!!
Gast11022013 Auf diesen Beitrag antworten »
RE: Aut=Inn
Okay, dankeschön!

PS. Sollten meine Fragen Dich reizen, weil sie vielleicht zu dümmlich sind, so tut mir das leid, aber nur durch Fragen wird man eventuell klüger, wenns auch vermeintlich dumme Fragen sind. Wink
Mystic Auf diesen Beitrag antworten »
RE: Aut=Inn
Was mich reizt ist höchstens, dass du bei allereinfachsten Dingen (Beispiel: Erhalten Gruppenhomorphismen generell die Ordnungen der Elemente?) nicht selbst ein wenig herumprobierst, bevor du diese Frage stellst... Du profitierst ja dann auch von einer Lösung viel mehr, wenn du nachher sagen kannst, aha, genau an dieser Stelle habe ich offenbar falsch gedacht und diesen Irrweg muss ich daher in Zukunft meiden... Augenzwinkern
tigerbine Auf diesen Beitrag antworten »
Erweiterte Fragen.
Hallo Mystic,

wenn du auch noch einen Moment für mich Zeit hättest? Ich frage mich, warum man hier n=3 gewählt hat.

Zitat:
Richtig wäre:



Hauptaufwand liegt bei mir in der Bestimmung des ersten "=". Könnte man nun direkt nachrechen oder die Isomorphie benutzen. Aufwand liegt dann in der Bestimmung des Zentrums.



Für n=1 folgt sofort. Für n=2 auch, allerdings ist hier das Zentrum nicht trivial (nachrechnen oder prim verwenden). Für alle n>2 ist die Gruppenordnung nicht mehr prim. Kann man mit dem Ergebnis für n=3 folgern, dass für alle n>3 das Zentrum auch trivial ist? Ich denke da so in Richtung "Einbettung der in die ", kann es aber noch nicht in Form bringen.

Wenn nicht trivial ist, dann gibt es mit insbesondere . Nun hänge ich, dass ich nur hab und ich bräuchte ja für den Widerspruch.

Vielleicht ist die Idee aber auch schon falsch. Augenzwinkern
Mystic Auf diesen Beitrag antworten »
RE: Erweiterte Fragen.
Zitat:
Original von tigerbine
Aufwand liegt dann in der Bestimmung des Zentrums.



Für n=1 folgt sofort. Für n=2 auch, allerdings ist hier das Zentrum nicht trivial (nachrechnen oder prim verwenden). Für alle n>2 ist die Gruppenordnung nicht mehr prim. Kann man mit dem Ergebnis für n=3 folgern, dass für alle n>3 das Zentrum auch trivial ist? Ich denke da so in Richtung "Einbettung der in die ", kann es aber noch nicht in Form bringen.

Ja, das geht, der Beweis, dass das Zentrum für trivial ist lässt sich unmittelbar auch für die Symmetrischen Gruppen verallgemeinern, sodass n=2 die einzige Ausnahme bleibt...

Zitat:
Original von tigerbine
Wenn nicht trivial ist, dann gibt es mit insbesondere . Nun hänge ich, dass ich nur hab und ich bräuchte ja für den Widerspruch.

Vielleicht ist die Idee aber auch schon falsch. Augenzwinkern

Ich sehe nicht, dass diese Idee zielführend wäre... Wie gesagt, auch für den allgemeinen Beweis in sind nur 3 verschieden Elemente involviert, man muss sich den Fall n=3 genau ansehen und verallgemeinern...
tigerbine Auf diesen Beitrag antworten »
RE: Erweiterte Fragen.
In einem anderen Thread hattest du mir was über die Gestalt von beigebracht. Bleiben wir in der .



=> Man hat nur 2 Möglichkeiten für , dann ist schon festgelegt. Die Identität und

Testen wir nun die zweite Variante mit einem anderen Element der :



Es muss also gelten . Betrachten wir nun die . Auch dort finden dir das obige Beispiel. Damit ist dann aber doch auch schon festgelegt. Das müßte man nun Induktiv weitermachen.

Direkt könnte ich mir vorstellen, obiges Beispiel für die Paarungen (12), (132) - (23),(243) - (n-1,n)(n-2,n,n-1) zu betrachten und daraus zu folgern, dass das Zentrum trivial ist. Erstaunt2
Mystic Auf diesen Beitrag antworten »
RE: Erweiterte Fragen.
Ich hätte das ungefähr so bewiesen... Es sei die symmetrische Gruppe für ein n . Angenommen es gibt ein



welches im Zentrum von liegt.

Dann wählen wir so, dass . Da insbesondere mit vertauschbar ist und Konjugieren mit die Transposition liefert, muss sein. Wir wählen nun ein weiteres Element und untersuchen die Vertauschbarkeit von mit .. Durch Konjugieren erhalten mit erhalten wir , das kann aber unmöglich (x y z) sein, Widerspruch!
tigerbine Auf diesen Beitrag antworten »
RE: Erweiterte Fragen.
Hatte deinen vorherigen Post so verstanden, dass ich den Widerspruchsansatz schon allgemein nicht weiter verfolgen sollte. Was aber nciht heißt, dass ich ansonsten auf deinen Weg gekommen wäre. Freude
Mystic Auf diesen Beitrag antworten »
RE: Erweiterte Fragen.
Ja, vielleicht habe ich mich auch nicht deutlich genug ausgedrückt... Augenzwinkern

Auf jeden Fall sind nur drei verschiedene Elemente x,y,z in dem ganzen Beweis involviert, d.h., es spielt sich alles in der zu isomorphen Untergruppe ab, welche von den Permutationen auf {x,y,z} erzeugt wird...
jester. Auf diesen Beitrag antworten »

Ich möchte kurz etwas anmerken, was ich gerade beim "Stöbern" entdeckt habe.

Es ist zwar richtig, dass für , jedoch gibt es noch einen Fall, wo , nämlich . Dort gilt und . Es gibt also einen äußeren Automorphismus, der in dem entsprechenden Wikipediaartikel treffenderweise auch als "exotisch" bezeichnet wird: http://en.wikipedia.org/wiki/Automorphis...ernating_groups
Neue Frage »
Antworten »



Verwandte Themen